Limit at infinity of factorial and exponential function

แชร์
ฝัง
  • เผยแพร่เมื่อ 2 พ.ย. 2023
  • In this video, I showed how to compute the limit of a rational function of factorial and exponential composition

ความคิดเห็น • 80

  • @ECela-yw9sq
    @ECela-yw9sq 9 หลายเดือนก่อน +8

    Love your chalks and your math, keep up with the good work!

  • @josephmartos
    @josephmartos 4 หลายเดือนก่อน +1

    Your way of explaining this was so beautiful :)

  • @Arkapravo
    @Arkapravo 5 หลายเดือนก่อน +4

    Cool method! You are a great teacher.

  • @KahlieNiven
    @KahlieNiven 5 หลายเดือนก่อน +2

    Great video to teach how to formalize an intuition.

  • @avi6n
    @avi6n 6 หลายเดือนก่อน

    Incredibly helpful, thank you 🙏

  • @davidcarras4743
    @davidcarras4743 9 หลายเดือนก่อน

    Another great video!

  • @nothingbutmathproofs7150
    @nothingbutmathproofs7150 หลายเดือนก่อน +1

    Sweet method. I really liked it.

  • @weslinpenacamacho1075
    @weslinpenacamacho1075 8 หลายเดือนก่อน

    I really like this exercise, thanks

  • @ThetrueIdiot6969
    @ThetrueIdiot6969 8 หลายเดือนก่อน +2

    i wonder why these educational videos get no views, even a stupid game play video can get higher view( those videos aren't fun or educational at all ) your content deserves more views

  • @cliffordabrahamonyedikachi8175
    @cliffordabrahamonyedikachi8175 6 หลายเดือนก่อน +2

    You are doing well.

  • @user-qh1lx9pw9f
    @user-qh1lx9pw9f 9 หลายเดือนก่อน +7

    Your teaching is excellent and I tell you to make videos about formal proof of limits and continuity involving radicals and denominator

    • @PrimeNewtons
      @PrimeNewtons  9 หลายเดือนก่อน +3

      I already have those videos. Thanks

    • @Orillians
      @Orillians 9 หลายเดือนก่อน

      BRO UR TOO GOOD AT TEACING MAN. @@PrimeNewtons

  • @nicolas.montero4941
    @nicolas.montero4941 หลายเดือนก่อน

    you are the man

  • @isidorolorenzo802
    @isidorolorenzo802 8 หลายเดือนก่อน +1

    Just a smooth remark, my dear friend. It's a matter of notation: as infinity is not a number (except in the dual space), you can't stand a limit equal to infinity but tending to infinity. 😏👍

  • @michaelstahl1515
    @michaelstahl1515 6 หลายเดือนก่อน

    Nice proof , really !

  • @saidnajarro
    @saidnajarro 4 หลายเดือนก่อน

    I love you!

  • @John-uo5tm
    @John-uo5tm หลายเดือนก่อน

    Cool limit! For some real fun, replace the 3 with n. 😊

  • @user-rq3tl1nk8r
    @user-rq3tl1nk8r 3 หลายเดือนก่อน

    very very nice.

  • @aguyontheinternet8436
    @aguyontheinternet8436 9 หลายเดือนก่อน +7

    I mean, imagine the fraction n!/(3^n). What does it equal, who knows, just needs to be greater than 0. Now look at the fraction when you increase n by 1
    (n!)/(3^n) * (n+1)(3)
    So for any n>3, this function will grow larger, making the limit infinity as n gets larger and larger.
    Edit: watching the video, that was the exact thing you took advantage of

  • @TheMasterGreen
    @TheMasterGreen 4 หลายเดือนก่อน +1

    correct me if I am wrong but isn't it obvious that the numerator is clearly greater than the denominator when n --> inf therefore implying that it approaches +infinity? Or am I wrong?

  • @davidbrisbane7206
    @davidbrisbane7206 7 หลายเดือนก่อน

    Note ... 6^n/3^n < n!/3^n, where n > = 9.
    Now lim (6^n)/(3^n), as n goes to infinity = limit 2^n, as n goes to infinity, and this limit is infinite, hence the limit ( n!/3^), as n goes to infinity is also infinity .

  • @italixgaming915
    @italixgaming915 9 หลายเดือนก่อน +1

    To make your proof perfectly clean, you should write that you consider the case n>5 otherwise the part you're deleting doesn't exist.
    Anyway, you don't need to develop everything.
    Let's call a(n)=n!/3^n.
    You have: a(n+1)/a(n)=[(n+1)!/n!].[3^n/3^(n+1)]=(n+1)/3.
    So a(n) is a rising series.
    You can also say that a(n+1)=a(n).(n+1)/3 => a(n+1)-a(n)=a(n)/3 and since a(n) is rising, so does a(n+1)-a(n).
    But by the definition of the limit, a series can't converge if the difference between its consecutive values becomes greater and greater. So the series diverges. And since it's rising, its limit must be +inf.
    Last method, even quicker:
    The answer was on your tee-shirt from the beginning.

  • @roddos
    @roddos 6 หลายเดือนก่อน

    Great.

  • @GreenMeansGOF
    @GreenMeansGOF 3 หลายเดือนก่อน

    I usually use Stirling’s formula for these kinds if problems but this is better, especially if we have to prove the limit by definition.

  • @steftetane
    @steftetane 9 หลายเดือนก่อน +3

    Thanks for another great math video. What about the limit of n!/n^n ?

    • @hydroarx
      @hydroarx 9 หลายเดือนก่อน +11

      n^n = n×n×n×n×n×...
      Whereas
      n! = n×(n-1)×(n-2)×(n-3)×...
      n^n is clearly bigger
      So this limit approaches 0

  • @samyaksharma2550
    @samyaksharma2550 8 หลายเดือนก่อน

    ratio test is saviour

  • @AbouTaim-Lille
    @AbouTaim-Lille 6 หลายเดือนก่อน

    The factorial function is stronger than any power function a^x ,a>1.
    It can be shown if we calculate the limit of I(n) = x(n)/x(n+1),were x(n) = n!/a^n, a>1.
    We have : I(n) = (n!/ a^n )/ (n+1)!/a^n+1 = n!/(n+1)!. a = a/n+1 -->0 when n-->∞. Which makes it easy to see that x(n) is strongly decreasing even for the large values of a.
    To be honest the product a^n. n! is compared with n^n.

  • @kanizfatema6907
    @kanizfatema6907 9 หลายเดือนก่อน

    Sir , kindly make a full course on limit and calculas😢😢😢😢

  • @brunoporcu3207
    @brunoporcu3207 7 หลายเดือนก่อน

    Sorry, I dont understand the equal. If I delete that terms it shouldn't just be major?
    Thank you so much

  • @bradfoster6092
    @bradfoster6092 9 หลายเดือนก่อน

    Hell yea!

  • @colina64
    @colina64 9 หลายเดือนก่อน

    COOL

  • @bogusawsroda3747
    @bogusawsroda3747 6 หลายเดือนก่อน

    1:10 nice man

  • @ayanahmed5114
    @ayanahmed5114 9 หลายเดือนก่อน

    is the factorial function differentiable

    • @davidg5898
      @davidg5898 9 หลายเดือนก่อน +4

      It is not. Only continuous functions are differentiable. The factorial function fails that test because it is only defined for integers (non-negative integers, specifically).
      You can do a bit of mathemagic to come up with something akin to what you're asking by taking the derivative of the gamma function (a continuous version of the factorial function) and evaluating it at integer values.

  • @juergenilse3259
    @juergenilse3259 9 หลายเดือนก่อน

    I would look at the cases wit n>6:
    n!/3^n=(1*2*3*3*5*6)/3^6*product(i)/product(3) i=7 to n
    Let C=(1*2*3*4*5*6)/3^6
    n!/3^n=C*product(i)/product(3) i=7 to n > C*product(6)/product(3) i=7 to n =
    C*product(6/3) i=7 to n = C*(6/3)^n = C*2^n
    So lim(n!/3^n) > lim(C*2^n) = C*li(2^n) = infinity
    As you can see, lim(n!/3^n) diverges at least as fast as C*lim(2^n) with some constant C>0. In the video, it was shown, that lim(n!/3^n) diverges faster than lim(1/27*n), but in fact, it diverges not only faster than a linear term, it diverges even faster than an exponential term with base larger than 1.

  • @guest2649
    @guest2649 หลายเดือนก่อน

    i just thought of it like
    3^n = 3*3*3..
    while as
    n! = 1*2*3*4*5..
    So as only 2 products are less than the 3^n product, n! would take over 3^n quite fast

  • @roger7341
    @roger7341 7 หลายเดือนก่อน

    Stirling's approximation states that ln(n!) is approximately equal to n*ln(n)-n
    Thus, ln(n!/3^n) is approximately n*ln(n)-n-n*ln3=n[ln(n)-1-ln3], which tends to ∞ as n→∞
    Therefore n!/3^n→∞ as n→∞

    • @timetraveller2818
      @timetraveller2818 7 หลายเดือนก่อน

      a neater answer would be to probably to prove Stirling's approximation and then write just as you would say but this is also beautifully elegant
      Edit:Just realized that I didn't write to prove.

  • @dpmike32819
    @dpmike32819 9 หลายเดือนก่อน

    What about using the ratio test

    • @PrimeNewtons
      @PrimeNewtons  9 หลายเดือนก่อน +1

      Sadly, the topic this student was dealing with was limits. So I couldn't even mention Ratio test.

  • @jam9339
    @jam9339 9 หลายเดือนก่อน

    ♾️

  • @cameroncurtis7261
    @cameroncurtis7261 9 หลายเดือนก่อน +37

    Always remember, the only thing bigger than a factorial is another factorial 😅

    • @Cubowave
      @Cubowave 9 หลายเดือนก่อน +14

      Tetration: 👁️

    • @juergenilse3259
      @juergenilse3259 9 หลายเดือนก่อน +2

      I beg to differ: I think, the "ackermann function" is growing much faster than simple factorial ...
      en.wikipedia.org/wiki/Ackermann_function
      😇

    • @btb2954
      @btb2954 9 หลายเดือนก่อน

      x^x

    • @taito404
      @taito404 9 หลายเดือนก่อน +3

      ​@@Cubowave Pentation🗿

    • @Cubowave
      @Cubowave 9 หลายเดือนก่อน +1

      @@taito404 septation: (idk if that exists)

  • @chendingbang1536
    @chendingbang1536 9 หลายเดือนก่อน

    stirling approximation!

  • @dpmike32819
    @dpmike32819 9 หลายเดือนก่อน

    What about the ratio test? Easier

  • @vp_arth
    @vp_arth 6 หลายเดือนก่อน

    That limit is just 2 * (3/3) * (4/3) * (5/3) * infinite serie of more than one members

  • @WillBillDillPickle
    @WillBillDillPickle 5 หลายเดือนก่อน +1

    Who else is here from O'leary's Infinite Series Sheet?

    • @mathbrah
      @mathbrah 5 หลายเดือนก่อน +1

      me lol

  • @user-ud1zv2yh3r
    @user-ud1zv2yh3r 9 หลายเดือนก่อน +1

    This is so obvious. How could this take more than ten seconds? Let fsubn(n) = (n/3) * fsubn-1). This clearly diverges as n gets big.

  • @swampfolk2526
    @swampfolk2526 7 หลายเดือนก่อน

    Ну вообще этот предел очевиден без просмотра видео. Факториал каждым следующим N домножает на N предыдущий результат, а знаменатель каждое следующее N домножается на 3. очевидно что при бесконечности множитель будет последний как бесконечность деленная на 3, что есть бесконечность. Т.е. предел бесконечный очевидно.

  • @maciejterakowski9062
    @maciejterakowski9062 6 หลายเดือนก่อน

    I need 3 sec. to find the result

  • @ivandeneriev7500
    @ivandeneriev7500 9 หลายเดือนก่อน

    X!=840 how to solve (not graphically)

    • @rubixmc7320
      @rubixmc7320 9 หลายเดือนก่อน

      Use the gamma function. It is an integral, so you’ll need to look it up. Γ(n) = (n-1)! So just solve Γ(n+1) = 840

    • @Yougottacryforthis
      @Yougottacryforthis 8 หลายเดือนก่อน

      No solution

  • @user-ym9od7nn4d
    @user-ym9od7nn4d 6 หลายเดือนก่อน

    esta muito rápido sem explicar

  • @arolimarcellinus8541
    @arolimarcellinus8541 5 หลายเดือนก่อน

    why lots of limit problems need to assume that HS student passed their analysis class?? No one ever take mathematical analysis, but those limits problems force us to use it if we cannot use L'Hospital.
    Limit problems is the source of why students hate math. Because math teacher doesn't even teach analysis properly, especially in inequalities. Can you imagine how many students in the class that can make the inequality postulate (n!)/3^n >= (1*2*n)/(3*3*3)?? I can bet my 100 bucks it will be less than 10 in regular classes.

  • @dmihovilovic
    @dmihovilovic 6 หลายเดือนก่อน

    This looks too complicated.....n! is n(n-1)(n-2)....*1, therefore for a very large n,, n! will look like n^n. Then n!/3^n = n^n/3^n = (n/3)^n that tends to infinity when n tends to infinity.

  • @koenigtiger6550
    @koenigtiger6550 หลายเดือนก่อน

    Ну это очень простая задача.
    Я её решил за несколько секунд в уме.

  • @alfredopi8699
    @alfredopi8699 7 หลายเดือนก่อน

    SI può ottenere lo stesso risultato più velocemente sostituendo n! con la Formula di Stirling valida per "n grande"

  • @mathbrah
    @mathbrah 5 หลายเดือนก่อน

    Who else is here from O'leary's Infinite Series Sheet?